Difference between revisions of "2015 AMC 12B Problems/Problem 5"

(Problem)
(Solution)
 
(2 intermediate revisions by the same user not shown)
Line 2: Line 2:
 
The Tigers beat the Sharks 2 out of the 3 times they played. They then played <math>N</math> more times, and the Sharks ended up winning at least 95% of all the games played. What is the minimum possible value for <math>N</math>?
 
The Tigers beat the Sharks 2 out of the 3 times they played. They then played <math>N</math> more times, and the Sharks ended up winning at least 95% of all the games played. What is the minimum possible value for <math>N</math>?
  
<math>\textbf{(A)}\; ? \qquad\textbf{(B)}\; ? \qquad\textbf{(C)}\; ? \qquad\textbf{(D)}\; ? \qquad\textbf{(E)}\; ?</math>
+
<math>\textbf{(A)}\; 35 \qquad \textbf{(B)}\; 37 \qquad \textbf{(C)}\; 39 \qquad \textbf{(D)}\; 41 \qquad \textbf{(E)}\; 43</math>
  
 
==Solution==
 
==Solution==
 +
The ratio of the Shark's victories to games played is <math>\frac{1}{3}</math>. For <math>N</math> to be at its smallest, the Sharks must win all the subsequent games because <math>\frac{1}{3} < \frac{95}{100}</math>. Then we can write the equation
  
 +
<cmath>\frac{1+N}{3+N}=\frac{95}{100}=\frac{19}{20}</cmath>
 +
 +
Cross-multiplying yields <math>20(1+N)=19(3+N)</math>, and we find that <math>N=\fbox{\textbf{(B)}\; 37}</math>.
  
 
==See Also==
 
==See Also==
 
{{AMC12 box|year=2015|ab=B|num-a=6|num-b=4}}
 
{{AMC12 box|year=2015|ab=B|num-a=6|num-b=4}}
 
{{MAA Notice}}
 
{{MAA Notice}}

Latest revision as of 10:06, 4 March 2015

Problem

The Tigers beat the Sharks 2 out of the 3 times they played. They then played $N$ more times, and the Sharks ended up winning at least 95% of all the games played. What is the minimum possible value for $N$?

$\textbf{(A)}\; 35 \qquad  \textbf{(B)}\; 37 \qquad \textbf{(C)}\; 39 \qquad \textbf{(D)}\; 41 \qquad \textbf{(E)}\; 43$

Solution

The ratio of the Shark's victories to games played is $\frac{1}{3}$. For $N$ to be at its smallest, the Sharks must win all the subsequent games because $\frac{1}{3} < \frac{95}{100}$. Then we can write the equation

\[\frac{1+N}{3+N}=\frac{95}{100}=\frac{19}{20}\]

Cross-multiplying yields $20(1+N)=19(3+N)$, and we find that $N=\fbox{\textbf{(B)}\; 37}$.

See Also

2015 AMC 12B (ProblemsAnswer KeyResources)
Preceded by
Problem 4
Followed by
Problem 6
1 2 3 4 5 6 7 8 9 10 11 12 13 14 15 16 17 18 19 20 21 22 23 24 25
All AMC 12 Problems and Solutions

The problems on this page are copyrighted by the Mathematical Association of America's American Mathematics Competitions. AMC logo.png